• Anúncio Global
    Respostas
    Exibições
    Última mensagem

[LIMITE] Limite que tende ao infinito

[LIMITE] Limite que tende ao infinito

Mensagempor Mell » Qua Mai 08, 2013 00:09

Olá!! Resolvi o limite \lim_{\rightarrow\infty} \frac{\sqrt[7]{x^3-5x^2+1}}{x+3} da seguinte forma:

\lim_{\rightarrow\infty} \frac{\sqrt[7]{x^3-5x^2+1}}{x+3}=\frac{\sqrt[7]{x^3(1-\frac{5x^2}{x^3}+\frac{1}{x^3})}}{x(1+\frac{3}{x})} Considerei as parcelas \frac{5x^2}{x^3} , \frac{1}{x^3} e \frac{3}{x} como limites que tendem a zero e logo ficou assim:

= \frac{({x^3})^{1/7}}{x^1} = \lim_{\rightarrow\infty} {x}^{\frac{3}{7}-1} = {x}^{-\frac{4}{7}} = \lim_{x\rightarrow\infty} \frac{1}{{x}^{\frac{4}{7}}} = 0

Entretanto, me falaram que o resultado da questão era +\infty. Só que não consigo entender porque essa minha resolução está errada... Alguém pode ajudar?? Por favor sejam gentis rs talvez meu erro esteja óbvio para alguém, mas não consigo percebê-lo.
Mell
Novo Usuário
Novo Usuário
 
Mensagens: 8
Registrado em: Qua Mai 01, 2013 14:48
Localização: São Paulo, SP
Formação Escolar: GRADUAÇÃO
Andamento: cursando

Re: [LIMITE] Limite que tende ao infinito

Mensagempor e8group » Qua Mai 08, 2013 10:34

Sim está correto .Não faz sentido este limite resultar +\infty uma vez que a potência de x no denominador é estritamente maior que de x no numerador .
e8group
Colaborador Voluntário
Colaborador Voluntário
 
Mensagens: 1400
Registrado em: Sex Jun 01, 2012 12:10
Formação Escolar: GRADUAÇÃO
Área/Curso: Engenharia Elétrica
Andamento: cursando

Re: [LIMITE] Limite que tende ao infinito

Mensagempor Mell » Qua Mai 08, 2013 18:19

Está certo mesmo?!! (: Ah é porque tirei essa questão de uma prova que estava corrigida como certa com o resultado +infinito... mas não vi muito sentido na resolução e tentei fazer sozinha. Obrigada pela ajuda mais uma vez!
Mell
Novo Usuário
Novo Usuário
 
Mensagens: 8
Registrado em: Qua Mai 01, 2013 14:48
Localização: São Paulo, SP
Formação Escolar: GRADUAÇÃO
Andamento: cursando

Re: [LIMITE] Limite que tende ao infinito

Mensagempor e8group » Qua Mai 08, 2013 21:21

Mell escreveu:Está certo mesmo?!! (: Ah é porque tirei essa questão de uma prova que estava corrigida como certa com o resultado +infinito... mas não vi muito sentido na resolução e tentei fazer sozinha. Obrigada pela ajuda mais uma vez!


De nada .Sim está correto ,caso contrário eu e vc + o site wolfram alpha estaria errado .Pois http://www.wolframalpha.com/input/?i=li ... +infinity+ .
e8group
Colaborador Voluntário
Colaborador Voluntário
 
Mensagens: 1400
Registrado em: Sex Jun 01, 2012 12:10
Formação Escolar: GRADUAÇÃO
Área/Curso: Engenharia Elétrica
Andamento: cursando


Voltar para Cálculo: Limites, Derivadas e Integrais

 



  • Tópicos relacionados
    Respostas
    Exibições
    Última mensagem

Quem está online

Usuários navegando neste fórum: Nenhum usuário registrado e 105 visitantes

 



Assunto: Taxa de variação
Autor: felipe_ad - Ter Jun 29, 2010 19:44

Como resolvo uma questao desse tipo:

Uma usina de britagem produz pó de pedra, que ao ser depositado no solo, forma uma pilha cônica onde a altura é aproximadamente igual a 4/3 do raio da base.
(a) Determinar a razão de variação do volume em relação ao raio da base.
(b) Se o raio da base varia a uma taxa de 20 cm/s, qual a razão de variação do volume quando o raio mede 2 m?

A letra (a) consegui resolver e cheguei no resultado correto de \frac{4\pi{r}^{2}}{3}
Porem, nao consegui chegar a um resultado correto na letra (b). A resposta certa é 1,066\pi

Alguem me ajuda? Agradeço desde já.


Assunto: Taxa de variação
Autor: Elcioschin - Qua Jun 30, 2010 20:47

V = (1/3)*pi*r²*h ----> h = 4r/3

V = (1/3)*pi*r²*(4r/3) ----> V = (4*pi/9)*r³

Derivando:

dV/dr = (4*pi/9)*(3r²) -----> dV/dr = 4pi*r²/3

Para dr = 20 cm/s = 0,2 m/s e R = 2 m ----> dV/0,2 = (4*pi*2²)/3 ----> dV = (3,2/3)*pi ----> dV ~= 1,066*pi m³/s


Assunto: Taxa de variação
Autor: Guill - Ter Fev 21, 2012 21:17

Temos que o volume é dado por:

V = \frac{4\pi}{3}r^2


Temos, portanto, o volume em função do raio. Podemos diferenciar implicitamente ambos os lados da equação em função do tempo, para encontrar as derivadas em função do tempo:

\frac{dV}{dt} = \frac{8\pi.r}{3}.\frac{dr}{dt}


Sabendo que a taxa de variação do raio é 0,2 m/s e que queremos ataxa de variação do volume quando o raio for 2 m:

\frac{dV}{dt} = \frac{8\pi.2}{3}.\frac{2}{10}

\frac{dV}{dt} = \frac{16\pi}{15}